面白い問題おしえて〜 ..
[2ch|▼Menu]
163:132人目の素数さん
10/05/01 12:23:47
くだらねぇ問題はここへ書け ver.3.14(63桁略)7816
が沈んで消えたので転載

954 132人目の素数さん [sage] 2010/04/23(金) 14:29:44 ID: Be:
「数学・まだこんなことがわからない−素数の謎から森理論mで−」吉永良正(講談社ブルーバックス)
に載ってた問題です。

「(2^n + 1)/n^2 が整数となるような1より大きい整数nを全て決定せよ」
(答えは、「n=3」のみ。)

この証明方法を教えて欲しいです。
『フェルマーの小定理』の小定理を知っていないとまず解けない問題だそうです。
元々は、1990年の数学オリンピック北京大会で出題された問題ですが、日本勢で正解した人はいなかったそうです。
本では、これというのも日本の学校では初等整数論さえまともに教えてないから云々といった感じで続いていて
問題の解法には一切触れられていませんでした。

164:132人目の素数さん
10/05/01 12:24:48
(続き)
995 954 [sage] 2010/04/28(水) 15:52:01 ID: Be:
もう忘れられかけているので自己解答します。結局ググル先生に教えてもらいました。
テニオハがなっていませんが普通に理解できると思います。
適切な指導者がいて特訓すれば数学オリンピックなんて大した事ないのかもと思えてきました。
(天才は指導者がいなくても自力で成長できるんだろうけど・・・)

n>1 で n^2|2^n+1 が成り立つと仮定する。

(1). n|2^n+1よりnは奇数。nの最小素因数をpとする。p|2^n+1、すなわち2^n≡-1(mod p)。
2^i≡-1(mod p)となる最小の数をiとする。2^(p-1)≡1(mod p)より、i<(p-1)。
n=ki+r (0≦r<i)とおくと、2^n≡(-1)^k・2^r≡-1(mod p)。kは偶数だとすると、2^r≡-1(mod p)
となりiの選び方と矛盾するのでkは奇数。よって2^r≡1(mod p)。
2^(i-r)≡2^r・2^(i-r)≡2^i≡-1(mod p)かつiの最小性により、r=0。
i|n,i<(p-1)によりi=1。よって2≡-1(mod p)すなわちp|3、よってp=3。

(2). n=3^k・d, (d,3)=1とする。まずk≧2 と仮定する。n^2|2^n+1より、3^(k+2)|1-(1-3)^n。
よって、3^(k+2)|3^(k+1)・d- Σ[h=2,k+1]{C<n,h>・(-1)^h・3^h}。
h!に含まれる3の指数はh/2(=h/3+h/9+h/27+…)未満かつh≧2なので、3^(k+2)|C<n,h>・3^h。
これは、3|d となるので矛盾する。よってk=1、すなわちn=3d。

(3). d>1と仮定した上でdの最小素因数をqとする(q≧5)。 q|2^n+1すなわち2^n≡-1(mod q)。
2^j≡-1(mod q)となる最小の数をjとする。2^(q-1)≡1(mod q)より、i<(q-1)。
((1)と同様なので中略)、j|n。 またqはdの最小素因数であり,j<q-1,nは奇数。
よってj=1またはj=3、すなわちq|3またはq|9。どちらもq=3となりq≧5に矛盾する、よってd=1。

以上により、n>1 の場合の候補は3のみ。
n=3の時に成り立つのは明らか。[証明終了]


165:132人目の素数さん
10/05/01 12:26:20
↑自分で書き込んでおいてなんだけど、結構気に入っている解法です。
もっと簡単またはエレガントな解法はあるでしょうか?

166:132人目の素数さん
10/05/04 00:56:26
>>164

(1) の補足
 2^(p-1)≡1 (mod p)     (← フェルマーの小定理)
 n = ki + r, (0≦r<i)       (← nをiで割った)
 i|n とpの定義より i=1 または i≧p,
 i<p-1 より i=1,

(2) の補足
 2^n + 1 = (3-1)^2 + 1
     = Σ(h=1,n) C(n,h)・(-1)^(n-k)・3^h    (←二項定理)
     = 3^(k+1)・d - Σ(h=2,k+1) C(n,h)・(-3)^h + N・3^(k+2),    (n:奇数)
 h≧2 のとき C(n,h) = n(n-1)・・・(n-h+1)/h! = (3^k)d・(n-1)・・・・・(n-h+1)/h!,
 h! の素因数分解における3の冪指数は h/2 未満。    (←補題)
 ∴ C(n,h)・3^h 中の3の冪指数は k-(h/2)+h = k+(h/2) ≧ k+1 より大きい。
 ∴ 3^(k+2) | C(n,h)・3^h
 題意より 3^(k+2) | 3^(2k) | (2^n + 1),
 ∴ 3^(k+2) | 3^(k+1)・d
 ∴ 3 | d   (dの定義に矛盾)

〔補題〕 h! の素因数分解におけるpの冪指数I(h,p)は h/(p-1) 未満。
 I(h,p) = Σ(e=1,h) [h/(p^e)] ≦ Σ(e=1,h) h/(p^e) = {h/(p-1)}{1 - 1/(p^h)} < h/(p-1), (終)


〔参考書〕
秋山 仁+ピーター・フランクル 共著「完全攻略 数学オリンピック」日本評論社 (1991/11/20)
 ISBN 4-535-78185-0   p.68〜70 (ミスプリント有り)

URLリンク(www.math.s.chiba-u.ac.jp)
 第1回(1959) 〜 第40回(1999)の問題

167:132人目の素数さん
10/05/04 01:51:56
(1)の「nの最小素因数をpとする」を「nの素因数の1つをpとする」に書き換えれば
簡単になるのでは。

168:132人目の素数さん
10/05/04 20:30:48
hoge

169:132人目の素数さん
10/05/04 20:46:34
>>165

〔補題〕 gcd(q^a -1, q^b -1) = q^gcd(a,b) - 1,   ……… (☆)
を利用する手もある。

(ヒント) ユークリッドの互除法で…

〔参考書〕
 秋山 仁+ピーター・フランクル (共著) 「数学オリンピック全問題 1984〜1990」日本評論社 (1990/09/10)
 ISBN 4535781869 p.118〜119


>>167
i|n, i<p-1 から i=1 を出ませぬ〜〜

170:165
10/05/05 16:01:51
>>166 補足してくれてありがとう

>>168
自分なりに解いてみました。面白いです。
ただ補題の結果(+導出過程)を事前に知っていないと思いつくのはかなり難しいかも
これは有名な定理?何か名前が付いているのでしょうか?

〔補題〕 (q^a-1, q^b-1)=q^(a,b)-1
f(x)=x^a-1, g(x)=x^b-1 と置き、最大公約多項式 (f(x), g(x)) を h(x) と置く。
代数式f(x)=0 と g(x)=0 の解集合は其々位数 a,b の巡回群を形成する。(中略{真面目に書くと長い・・・}) h(x)=x^(a,b)-1。
ユークリッドの互除法により、α(x)・f(x)+β(x)・g(x)=h(x) (α(x),β(x)は適当な多項式)、
即ち α(x)・f(x)/h(x)+β(x)・g(x)/h(x)=1 と表せる事から、x に q を代入すると、(f(q)/h(q), g(q)/h(q))=1 つまり互いに素である。
よって、(q^a-1, q^b-1) = (f(q), g(q)) = (f(q)/h(q)・h(q), g(q)/h(q)・h(q)) = h(q) = q^(a,b)-1。[補題の証明終了]

〔本題〕
n>1 で n^2|2^n+1 が成り立つと仮定する。 nが奇数なのは明らか。
a,bが共に奇数の場合は、補題の証明途中で x に -q を代入することにより、関係式 (q^a+1, q^b+1) = q^(a,b)+1 が得られる。
ここで q=2, a=3, b=n と置けば、(9, 2^n+1) = 2^(9,n)+1。
(9,n)=1 の場合は、(3,n)=1。これは >165 の(1)の結果と矛盾するのでありえない。
(9,n)=3 の場合は、n=3d、但し(3,d)=1。以降は >165 の(3)と同じ。
(9,n)=9 の場合は、9≧(9, 2^n+1)=2^9+1=513>9 となるのでありえない。
よって、n=3 の場合のみ成り立つ。[証明終了]

>165 の(2)での二項定理の部分(たぶん一番の山場)が省けました。

171:165
10/05/05 19:36:04
> f(x)=x^a-1, g(x)=x^b-1 と置き、最大公約多項式 (f(x), g(x)) を h(x) と置く。
> 代数式f(x)=0 と g(x)=0 の解集合は其々位数 a,b の巡回群を形成する。(中略{真面目に書くと長い・・・}) h(x)=x^(a,b)-1。

↑あとで見直してみたら、この部分の証明は小難しく考える必要は無かったですねw

f(x)=x^a-1, g(x)=x^b-1, h(x)=x^(a,b)-1 と置く。
ユークリッドの互除法により、sa +tb=(a,b) と書ける。ただしs,tは適当な整数係数
連立式 f(x)=0,g(x)=0 の任意の解ζについて、ζ^(a,b) = ζ^(sa +tb) = 1^s ・ 1^t = 1。 すなわち h(x)=0 の解である。
またh(x)=0 の任意の解ξについては、ξ^a = ξ^{(a,b)・a/(a,b)} = 1^{a/(a,b)} = 1、 ξ^b = ξ^{(a,b)・b/(a,b)} = 1^{b/(a,b)} = 1。すなわち 連立式 f(x)=0,g(x)=0 の解である。
以上の結果と各多項式の(複素数)一次式分解を考慮すれば、(f(x), g(x))=h(x) は明らか。 [証明終了]

172:165
10/05/05 21:25:58
>170 の訂正・・・あぁぁ・・・

[間違い]
> ここで q=2, a=3, b=n と置けば、(9, 2^n+1) = 2^(9,n)+1。
> (9,n)=1 の場合は、(3,n)=1。これは >165 の(1)の結果と矛盾するのでありえない。
> (9,n)=3 の場合は、n=3d、但し(3,d)=1。以降は >165 の(3)と同じ。
> (9,n)=9 の場合は、9≧(9, 2^n+1)=2^9+1=513>9 となるのでありえない。

[訂正]
ここで q=2, a=3, b=n と置けば、(9, 2^n+1) = 2^(3,n)+1。
(3,n)=1 の場合は、>165 の(1)の結果と矛盾するのでありえない。
(3,n)=3 の場合は、n=3d、但し(3,d)=1 と置ける。以降は >165 の(3)と同じ。

173:165
10/05/05 22:00:19
しつこくてすみません。

> (3,n)=3 の場合は、n=3d、但し(3,d)=1 と置ける。以降は >165 の(3)と同じ。

[再訂正]
(3,n)=3 の場合は、n=3^k・d、但しk≧1, (3,d)=1 〜〜〜

んー、結局 >165 の(1), (2), (3) が全部必要になってしまう・・・。

174:132人目の素数さん
10/05/09 20:22:35
平成教育委員会で出た
「円の中に3つの同じ長さの線(直線でなくてもいい)を引いて10個の領域に分けろ」
って問題が答えが何でもアリすぎて別の意味で面白かったsage

175:132人目の素数さん
10/05/09 21:45:34
覚えがないなー
8の字をもっとつなげた曲線1個で何個でも分けれるんじゃないの?

176:132人目の素数さん
10/05/14 14:01:46
このスレのurlに0が多いのが一番面白い

177:132人目の素数さん
10/05/14 21:16:02
x(1)、x(2)、…、x(n)を自然数として、n変数関数f(x(1), x(2), …, x(n))=Σ[k=1,n]x(k)が f< 1を満たす範囲で最大値を取るとき、x(k)は平方数にならない。
○か×か

178:132人目の素数さん
10/05/15 06:07:17
Σ[k=1,n]x(k)が f< 1

179:132人目の素数さん
10/05/15 10:46:55

職場の同僚が、ガソリン価格が、値上がり中に満タンにするのと、値下がり中に満タンにするの
とでは、値上がりに、中に満タンにするほうが得だと言う。
そいつが、自信有りげに言うんで色々ググッたけどわからなかったんで、微積得意な人がいたら
本当かウソか、わかりやすく教えてもらえないだろうか?

スレチだったらスマン。


180:132人目の素数さん
10/05/15 12:29:05
>>163
以前考えたアイデア。
誰かのと重複してるかも。

-------------------------------------------------------------------------------
【n=3^m の場合の nCi・3^i (i≧2) が 3^(m+2) で割り切れることの証明】

( i !の 3のべき数) < (i/3)+(i/3^2)+...= i/2 より
( nCi の 3のべき数) > m−i/2
よって i≧2 のとき ( nCi・3^i の 3のべき数) > m−i/2+i=m+i/2≧m+1より
( nCi・3^i の 3のべき数) ≧m+2
-------------------------------------------------------------------------------
【n=k・3^m の場合】

n は奇数で 3 の倍数の必要があり、
n=(2k+1)・3^m (2k+1not≡0 (mod 3)) とかけます。(not≡ は合同でないの意味です)
このとき、p=3^m として
(2^n+1)/n^2={ ( 2^p)^(2k+1)+1}/{ (2k+1)^2・p^2 }
( 2^p)^(2k+1)+1=(2^p+1) { (2^p)^(2k)−(2^p)^(2k-1)+...−2^p+1}
ここで (2^p)^(2k)≡−(2^p)^(2k-1)≡...≡−2^p≡1 (mod 3) より
(2^p)^(2k)−(2^p)^(2k-1)+...−2^p+1≡2k+1≡not≡0 (mod 3)
よって、
(2^n+1)/n^2 が整数 ⇒ { 2^(3^m)+1} /(3^m)^2 が整数


181:132人目の素数さん
10/05/15 13:44:28
>>179
値上がり中→最高値→値下がり中→最安値→値上がり中→最高値→の繰り返しだとして
一番いいのは最安値のときだけどその時点で最安値かはわからない
まだ下がるかもしれないし上がるかもしれない
だから値上げに転じた時点でガソリン満タンにするのがいい

182:132人目の素数さん
10/05/15 15:47:07
>>181

どうもありがとう m(_ _)m

183:132人目の素数さん
10/05/20 02:40:03
3つの実数x,y,z ( x+y+z > 0 , y < 0 )を (x,y,z) → (x+y,−y,z+y)
とする変換を考えます。変換後、負の数のどれか(存在するとき)をyとして残りをx、zと
して同じ変換を繰り返します。このとき変換のときにどのように負の数を選んでも
必ず有限回の変換後に3つの数すべてが非負実数になることを証明せよ。

184:132人目の素数さん
10/05/20 23:47:57
>>183
IMOに似た問題が無かったっけか

185:132人目の素数さん
10/05/21 00:54:10
Art of Problem Solving ? International Competitions IMO
URLリンク(www.artofproblemsolving.com)
どれのことだろう?

186:132人目の素数さん
10/05/21 08:45:41
時計の時針・分針・秒針の全てが同じ長さ・同じ重さだったとする。


時計が一番つらい時間は何時何分何秒か?

187:132人目の素数さん
10/05/21 09:09:24
つらい時間wwww
3時15分15秒あたりなんだろうがおもしろい問題だww

188:132人目の素数さん
10/05/21 12:30:24
>>186
便乗

3つの針がどれも重なっていないのに
針の区別が付かない時刻はあるか?
針は連続に動くものとする。

189:132人目の素数さん
10/05/21 15:43:29
ある日蛙が深さ30フィートの井戸の中に落ちてしまいました
蛙は昼の間3フィート井戸の壁をのぼる事ができ、夜の間2フィート下がってしまいます
蛙が井戸を抜け出すのに何日かかるでしょう?

190:132人目の素数さん
10/05/21 17:40:44
>>188
いっぱいあるくない?
2つの針の角の二等分線上に残りの針が来るとき

191:132人目の素数さん
10/05/21 23:39:30
186 だが、無意識に投稿した時間がかなり辛そうで自分で笑ってしまったw
あと4秒遅れてたら…
時計の死亡時刻ってだいたいこの辺に集中してるでしょ

>>187
3時15分15秒はふくらはぎが筋肉痛になる時間

192:132人目の素数さん
10/05/22 00:23:11
>>185
86年の3番が似ている

193:132人目の素数さん
10/05/22 01:21:31
>>190
なんでそれで区別が付かないの?

たとえば3時0分7.5秒ちょっとの時は 短針と長針の間に秒針が来るけど
3の針はその位置関係から区別が付くと思うんだが。

194:132人目の素数さん
10/05/23 03:03:36
1周の角度を 1、時針分針秒針を h, m, s とすると
それが「時計として正しい」とは、
12h-m が整数
60m-s が整数
が成り立っていることに同値。時計の公理、とでも名付けよう。

これから、「入れ替わりの時刻も正しい」は、式をいじると、
それぞれ以下の数が整数になることと同値となる。
(1) m⇔s : 43188h
(2) h⇔s : 518399h
(3) h⇔m : 43188h
(4) s→m m→h h→s : 414672h
(5) s→h h→m m→s : 576h

時計の定理1
(1)と(3)は見分けが付かないので、
時針のみ確定できること、または、秒針のみ確定できること、はどちらもあり得ない。

時計の定理2
これらのどの入れ替わりかも見分けが付かないのは 9283932178928064h が整数の時。
1日に 18567864357856128 回だけ起こる。


195:132人目の素数さん
10/05/23 03:29:16
>>188 にさらに便乗。
入れ替えてもばれない時刻に、
時計は、こっそり入れ替えてもばれない針同士を交換できるとする。

時計が「0時0分0秒を再度再現する」という
普段 12 時間かかってやっている仕事をサボろうとしたら、
最短で何分で達成できるか?

196:194
10/05/23 04:00:17
(3) を間違った。

(1) m⇔s : 43188h
(2) h⇔s : 518399h
(3) h⇔m : 143h
(4) s→m m→h h→s : 414672h
(5) s→h h→m m→s : 576h

時計の定理1
これらのどの入れ替わりかも見分けが付かないのは 1327602301586713100h が整数の時。
1日に 2655204603173426200 回だけ起こる。
200倍くらい増えた。

197:132人目の素数さん
10/05/23 11:35:14
>>196

> 3つの針がどれも重なっていないのに 


198:132人目の素数さん
10/05/23 12:06:50
>>196
> これらのどの入れ替わりかも見分けが付かないのは 1327602301586713100h が整数の時。 

どうしてそういう理屈になるん?

とりあえず、 hが 1/1327602301586713100 のとき 
mの位置は 60/1327602301586713100
s は 3600/1327602301586713100 だけど
この位置ではどの針が交換可能なんだろう?

オレがなにかかん違いしてる?

199:132人目の素数さん
10/05/23 22:17:29
>>196
hの針の位置が決まれば m, s の針の位置が自動で決まるので、m <=> s はありえない。

複素数浸かって、z = e^iθ を時針の位置にして、
(z, z^12, z^(12*60)) = (z1^12, z1, z1^(12*60))
(z, z^12, z^(12*60)) = (z2^12, z2^(12*60), z2)
(z, z^12, z^(12*60)) = (z3^(12*60), z3, z3^12)
(z, z^12, z^(12*60)) = (z4^(12*60), z4^12, z4)
を計算すればいいんじゃないだろうか、計算していないけど。

200:194
10/05/24 23:03:16
なんか全般的に間違ったな。
正しい時計であるために
・12h-m が整数 A
・720h-s が整数 B
ここまではいいとして、

重なる条件:
m=s:12h-A = 720h-B がなりたつとき → 708h = 2*2*3*59h が整数の時
h=m:12h-h = 11h が整数の時
h=s:720h-h = 719h が整数の時

交換できる条件:
m⇔s: h で m, s が一意に決まるので m=s の時以外はない。
s⇔h:
12s-m = 12(720h-B)-(12h-A) が整数 → 2*2*3*59h が整数
720s-h = 720h-(12h-A) が整数 → 8628h = 2*2*3*719h が整数
よって 2h、3h が整数のときのみ。
しかしそのときは必ず m=s となる
h⇔m:
12m-h = 12(12h-A)-h が整数 → 11*13h が整数
720m-s = 720(12h-A)-(720h-B) が整数 → 2^4*3^2*5*11h が整数
よって 11h が整数の時。しかしそのとき必ず h=m となる。
s→m m→h h→s:
12m-s = 12(12h-A)-(720h-B) が整数 → 2^6*3^2h が整数
720m-h = 720(12h-A)-h が整数 → 53*163h が整数
よって h が整数の時。しかしそのとき必ず m=s=h となる。
s→h h→m m→s:
12s-h = 12(720h-B)-h が整数 → 53*163h が整数
720s-m = 720(720h-B)-(12h-A) が整数 →2^2*3*13*3323 が整数
よって h が整数の時。しかしそのとき必ず m=s=h となる。

よって、重なるとダメとすると解なし?

201:132人目の素数さん
10/05/25 02:38:26
なるほど、時計の針の長さがちがうのは見易さのためであって
長さを違える必要はないのか


202:132人目の素数さん
10/05/25 11:27:51
(面白い問題かどうかわかりませんが、他に適してるスレが見当たらないので質問させてください)

塾講師してるんですが、
生徒が4人の一斉授業で、
4人は横一列に並んでます。

で、4人をできるだけ同確率で指名しつつ、
生徒としては、アットランダムに指名されてるように感じる、
そんな方法ってないでしょうか?

なお、時計を使うのはなしで。
(ありなら、現在秒を4で割ったあまりで、指名できますけど、
あんま時計みるわけにいかないので)
あと、連続で同じ人をあて「すぎる」のも避けたいです。
たまに「連続であてる」のなら、むしろそのほうがアットランダム(てかフェイント)で、
望ましいかもしれませんが。

「僕には、円周率の各桁を4で割ったあまり」しか思いつきません。
(でもいちいち円周率みてらんない)

フィボナッチ数列の各項を4で割ったあまり・・・てのはどうなんでしょう?
(思いつきですが)

203:132人目の素数さん
10/05/25 13:53:23
>>202
生徒の面前で4面体さいころを振る

204:132人目の素数さん
10/05/25 16:26:19
普通のサイコロでいい。鉛筆でもいい。
1〜4が出るまで振るだけ。

205:132人目の素数さん
10/05/25 16:47:49
振り直しはだれる。一発で決まる方がいい。

206:132人目の素数さん
10/05/25 20:18:08
板書した文字数%4で。

207:132人目の素数さん
10/05/25 23:01:42
分かってるか怪しそうな顔の奴に当てる

208:132人目の素数さん
10/05/25 23:07:11
四面体サイコロでなくても紙で四角い筒を作ればいい

209:132人目の素数さん
10/05/26 00:59:42
なにもふり直しをするひつようはない。
生徒に0〜3の番号をつけ
mod(先ほど当てた生徒+サイコロの目,4)とすれば十分だ

210:132人目の素数さん
10/05/26 10:03:06
>>186
掛け時計で文字盤の12を上にして一点でぶら下がってる状態だとして、
「一番つらい」の意味が力のモーメントが最大になるということだとすると
|sinθ+sin(12θ)+sin(720θ)|を最大にするθの値を求める必要があるな

211:132人目の素数さん
10/05/26 13:58:45
URLリンク(www.csicop.org)

212:132人目の素数さん
10/05/27 13:53:20
パズルの国のアリス
URLリンク(www.nikkei-science.com)
最新号に4月にやった問題の発展問題が載ってる

213:>>202
10/05/30 21:20:49
>>209
お、それいいっすね。
まあサイコロをふったり板書した文字の数数えるのは時間かかるので、
(脳内で一瞬で決めたいので)
てきとーな数字思い浮かべて、
mod 4か?とか思ったけど、
数理心理学(っていうの?)的に、かたよりがあるかなーーーとおもってて。。。
でも、3けたとか4けたの数字を4でわるのは少し時間かかるし・・・って思ってた・・・。

で、mod(先ほど当てた生徒+「1から100までからランダムで連想」,4)
だったら、まあ現実的ですかねえ?
===
mod(先ほど当てた生徒+「1から32までからランダムで連想」,4)
だと、人間の心理的に、偏りでる?
あ、でも、連想する数に偏りでても、「先ほど当てた生徒+」があるからいいのかな???あれ?わかんなくなってきた

214:132人目の素数さん
10/05/30 23:03:08
トランプを十分に切る
スートに生徒を対応させる
52回目の指名が近づくと確率がばれてゆくのが弱点

215:132人目の素数さん
10/05/31 01:02:26
>>214
引いたカードを毎回戻せばいい。

216:132人目の素数さん
10/05/31 01:06:40
>>213
連想する数字が偶数ばかりだったら、当たる生徒に偏りが出るなどの欠点はあると思うが
思いつく数字に多少の偏りがあっても、数と生徒を直接対応させるよりは
さっきの生徒+のほうが偏りにくいと思う。


217:132人目の素数さん
10/05/31 01:16:12
>>215
大数の法則で満足するならそれでもいいけれど…
可能な限り回数が違いすぎないことを保証したい場合はどうだろうと考えて
このような結果に…

218:>>213
10/05/31 08:42:45
>>216
>連想する数字が偶数ばかりだったら、当たる生徒に偏りが出るなどの欠点はあると思うが

そういえば自分、数字連想するときって、素数が多いようなきが。ということはほとんど奇数になってしまうわw

219:132人目の素数さん
10/05/31 09:56:32
>>217
> 可能な限り回数が違いすぎないことを保証したい場合
それならカードは4枚だろ

220:132人目の素数さん
10/05/31 09:57:59
>>218
奇数ばかりなのは、次に当たらない生徒を予想しやすくなるだけで
偏りはあまり生まれない。

221:132人目の素数さん
10/06/02 22:33:47
>>202 これ面白いなぁ
以前カジノの持ち込み不可の計算機類を持ちこまずに
身体と脳だけでルーレットやブラックジャックの類の必勝法を計算する方法を
考えていたことがあるけど、なんか似てる

222:132人目の素数さん
10/06/03 00:14:44
>>202
4枚のエースを生徒に割り当てる。
この4枚と1枚のジョーカーの計5枚を持ち、
「一番下のカードを上に乗せる」を繰り返しながら授業をする。
問題の時は一番上に来たカードの主を回答者に指名する。
ジョーカーが出た時はそのさらに下のカードの主を回答者に指名する。
回答が終わると毎回、他のカードの順はそのままで
回答者のカードをジョーカーの上に移動させる。
こうすると「一番当たってない人」がジョーカーの下にくるようになって、
その人は他の人の2倍の確率で当たるようになって均等性が比較的早く自己回復し、
その上「当たりにくい人」の候補が3人になって「俺は次当たらないだろ」的予想もできなくなる。

223:132人目の素数さん
10/06/03 01:49:21
曲面の鏡だけを使って、
通常の時計の秒針・分針・時針と全く同じ動きを
影で再現する日時計はどうすれば作れるか?

224:132人目の素数さん
10/06/03 02:00:24
>>210
つまり裏側に仕込んだギアかなにかを使って
相反する力の相殺をしてもいいというモデルかな?

それがOKだと、表の針と 180°の角をなして長さと密度が同じ「裏側の針」を仕込めば、
初期速度のみそれぞれ与えてあとは慣性の法則で時計は仕事をしなくてよくなるかな

225:132人目の素数さん
10/06/03 02:27:59
>>223
何の影で?

226:132人目の素数さん
10/06/03 02:28:39
>>223
問題の意図がよくわからん。 もうすこし条件等を詳しく。

227:132人目の素数さん
10/06/03 02:29:07
>>223
夜は影ができないから無理…だとただのとんちだな
でも他に思いつかない。どうするんだろ

228:132人目の素数さん
10/06/03 02:40:02
>>223
巨大なスリットを使って…だめかな

229:132人目の素数さん
10/06/03 08:30:44
>>224
う〜ん、なんとなくわかるが
その場合「裏側の針」の力のモーメントが
もともとの針の力のモーメントを打ち消すように働いてるんだよね

230:223
10/06/03 23:07:52
自作自演的に答えをひとつ。

巨大な3階建ての鏡のビルを作る。
3階の天井に小さな穴をひとつだけ空けて、
3階の床に光ファイバーの入口を敷き詰める。
2階でファイバーをくねらせて1階で時計を再現。
これはエレガントじゃないけどこれでとりあえずできるでしょ。

ただ太陽に視直径が存在して3階の光の点がそれほどくっきり出ないから秒針がぼやけるのが難点。
凹面鏡か何かで平行光線に補正するのが必要かも。

>>225 もちろん鏡の影

231:132人目の素数さん
10/06/04 11:22:34
全部鏡面仕上げした部品で機械時計を作ってあとは適当に。

232:132人目の素数さん
10/06/04 15:33:32
多胡輝も降参

233:132人目の素数さん
10/06/04 16:18:50
単純化のために春分秋分の日の日中だけに使える短針だけの時計で考える。
太陽は黄道を約12時間かけて半周するわけだよね。
その間光りの方向は半周しか変わらない
しかしその間に針は一周しなくてはならない。
影は光りの向きの反対にしかできないのだから
なんとかして光りの方向を一周させる必要があるてこと?
秒針まで考えたら720周も?

234:132人目の素数さん
10/06/04 23:48:46
URLリンク(bit.ly) のような "デバイス" を使えば、
ファイバー内の光の角度を 0 < x < arctan(1 - d/2√2h)-45°に制限できる。
h を大きくし d を小さくすることで任意の精度で 0 に近づけられる。
ファイバーの断面は正方形で、"デバイス" は正方形の1辺の厚みを持つ平たい形とする。
このデバイスを違う方向に2回通すことで、光は断面にほぼ垂直な成分のみとなる。

>>230 の「2階」から「鑑賞部屋」とは別部屋の1階にある、
このデバイスがたくさんある「デバイス部屋」に光を連れていき平行光線以外を消した後、
「鑑賞部屋」でプラネタリウムばりに時計を再現すればおk


235:234
10/06/05 00:08:45
完成した URLリンク(bit.ly)

236:132人目の素数さん
10/06/05 09:43:57
曲面の鏡「だけ」を使って、という問題じゃないのか?

237:132人目の素数さん
10/06/06 01:53:44
>>236 その前提内で解けてると思うが
光ファイバーもスリットも曲面の鏡のひとつだし

238:132人目の素数さん
10/06/06 08:40:08
実際の時計のように針が連続で動く必要はないのか?


239:132人目の素数さん
10/06/11 04:22:29
age

240:132人目の素数さん
10/06/17 23:01:13
>>238
・太陽は点で視直径 0 とする
・分針、時針との同時表示はとりあえずあきらめて秒針のみ
・秒針の表現も針の先の直系 0 の点だけでよい

とりあえずこの制約でいくと、
太陽→天井の1点スリット→地面に置いた斜め平面鏡→天井にある曲面鏡→地面
という構成で別解はありそう。

スリットを通った光が当たる曲面鏡上の点の座標を 0<t<T として
地面座標系 (x,y) にて
x=sin(43200t/T)
y=cos(43200t/T)
という位置に反射するように角度を構成すれば、連続も一応できる。

曲面鏡の連続性のために、
「曲面鏡は位置的に平面にあるけどその反射はさまざま」
のような配置はできないから、
z 方向にも波打たせて辻褄合わせないといけないけど。
43200個のコブがジグザグに並ぶ鏡面鏡が解になりそう。

241:240
10/06/17 23:05:53
・秒針・分針・時針、同時表示したい
なら、3点スリット使えばできそう。

・針を長さのある線分で表現したい
これを入れるには、スリットを線分にして、
角度と射影先の線分が映したい針に対応するようにすればいいけど、
曲面鏡の各点の角度と位置が微分可能な平面で3次元内の2次元曲面で構成できるかが
ちょっと分からない。

242:240
10/06/18 00:53:17
とりあえず、
天井の点スリット→地面に曲面鏡→天井の裏面を鏡じゃなくして、
点スリットを中心とする半径 c の位置にある点で秒針の先を表現
で考える。

URLリンク(bit.ly)
座標系は3次元直交座標右手系で
・点スリットが原点、
・地面方向が y 軸プラス、
・西方向と天井の時計の 12 時が x 軸プラス、
・北方向と天井の時計の 15 時が z 軸プラス。

太陽光のスリット点からの出射角を θ、
曲面鏡の光が落ちる表面の位置ベクトルを F(θ) = (r(θ)sinθ,0,0)、
F(θ)の点での曲鏡の単位法線ベクトルを N(θ)、
秒針 (second Hand) の先端の位置ベクトルを H(θ) とする。
秒針を再現するために H(θ) = (csin43200θ,ccos43200θ,0) 。

曲面鏡のベクトル方向の単位ベクトルを F'(θ)、
「鏡から秒針先端」のベクトル方向の単位ベクトルを H'(θ) とすると、
F'(θ) = F(θ)/|F(θ)|、
H'(θ) = (H(θ)-F(θ))/|H(θ)-F(θ)|、
N(θ) = (∂F/∂x)×(∂F/∂y)。

光と鏡の入射角と反射角の一致の法則から、
・F'(θ)・N(θ) = N(θ)・H'(θ)。
曲面鏡の微分可能性から、
・∃R∀θ(dr(θ)/dθ < R)
この2式を満たすr(θ)を求めればよい。

243:132人目の素数さん
10/06/18 16:55:01
その鏡って、その日用に設計されるもので翌日にはもう使えないんじゃね?
先のほうに夜は陽が出てないから無理というのがあったけど
夜使えないどころか日付限定用てこと?

244:132人目の素数さん
10/06/19 01:44:15
>>243
点スリットのさらに上に、
特定の1日だけその点スリットに光がくるように設計された円弧スリットを設ければ、
特定の1日だけ時計を表示することができる。
それを 365 日分作ってどれも同じ鑑賞部屋に照射するようにしておけばおk

245:132人目の素数さん
10/06/22 20:20:24
とある国のお話。

この国には無限の住人がおり、1番から順に、それぞれ自然数で番号付けされています。
この国の住人は皆、正直者か嘘つきのどちらかであり、
正直者は正しいことだけを言い、嘘つきは間違ったことだけを言います。

ある時、この国の住人たち全員が一斉に、次のような発言をしました。

「私より大きい番号の住人の中に、少なくとも1人嘘つきがいる」

さて、この事態について言えることは何でしょうか?

246:132人目の素数さん
10/06/22 20:44:00
むげにん

247:132人目の素数さん
10/06/22 22:53:51
>>245
出題者が 嘘吐きである。

248:132人目の素数さん
10/06/24 21:51:46
嘘つきのパラドックスは自己言及が原因だと思いきや、
>>245の場合、どの住人も自分以外の住人にしか言及しておらず、
間接的な自己言及も発生していない。
では、このパラドックスを生じさせているものは何か?

249:132人目の素数さん
10/06/24 23:23:18
パラドクスがおきているのか? 俺にはわからん。
どこで起きているのか具体的に指摘してもらえないだろうか?

250:132人目の素数さん
10/06/24 23:39:41
この世には少なくとも一人の嘘つきがいる

251:132人目の素数さん
10/06/25 23:53:35
>>245を実現する 正直者・嘘つき の配置は存在しないことを証明する。
1番の人間が嘘つきか正直者かで場合分けする。

1番が嘘つきなら、2番以降は全て正直者。…(1)
特に2番は正直者。よって、3番以降の人間で
嘘つきが居ることになる。これは(1)に矛盾。

1番が正直者なら、2番以降の住人の中に嘘つきが
少なくとも1人居る。i番が嘘つきだとすると、
i+1番以降は全て正直者。…(2)
特にi+1番は正直者。よって、i+2番以降の中に
嘘つきが少なくとも1人は居る。これは(2)に矛盾。

よって、どちらも起こりえない。すなわち、>>245の状況を
実現する 正直者・嘘つき の配置は存在しない。



パラドックスでも何でも無い。
解が存在しない方程式を提示してるだけ。

252:132人目の素数さん
10/06/26 09:41:59
パラドクスではなかったが
解が存在しない式であった。

253:132人目の素数さん
10/06/26 11:09:44
では数学的な形で書き直してみよう。

Nを1以上の自然数として、命題P(N)を次のように定義する。
「N+1以上の自然数nが存在して、P(n)は偽である」
P(1)が真だとしても偽だとしても、パラドックスが生じる。

これをパラドックスと言わないというのであれば、
パラドックスの定義が食い違ってる。

254:132人目の素数さん
10/06/26 11:28:33
一読して主張が明快な命題の場合にこそパラドックスの名が相応しい。
そうでなければ、単なる背理法により棄却される命題。



255:132人目の素数さん
10/06/26 11:33:13
>>253
> P(1)が真だとしても偽だとしても、パラドックスが生じる。 

すまんが 、P(1)が真だとして、 どこでパラドクスが生じるのかを
解説してもらえないだろうか?
できればP(1)が偽とした場合も。

どうもパラドクスの定義がくい違っているような気がしてならない。


256:132人目の素数さん
10/06/26 11:35:16
>>254
> 一読して主張が明快な命題の場合にこそパラドックスの名が相応しい。 

パラドクスの定義の話ならば、 そんなことはない。

パラドクスはそうあってほしいという願いならば
そのように(理解しやすい単純な形に))書き直してみてはどうだろうか? 


257:132人目の素数さん
10/06/26 12:21:33
>>255
>>251

258:132人目の素数さん
10/06/26 12:26:08
>>256
> そのように(理解しやすい単純な形に))書き直してみてはどうだろうか?
それがあなたの当為


259:132人目の素数さん
10/06/26 15:32:29
>>257の真意がわからない。

260:132人目の素数さん
10/06/26 15:35:05
>>258
>> パラドクスはそうあってほしいという願いならば 

そこの意味は
>>254がそのように願っている、という仮定の下において
 >>254書きなおしをに勧めている」 というものであって
願っているのは>>256ではないよ。


261:132人目の素数さん
10/06/26 16:27:32
逃げ口上だけはお上手。



262:132人目の素数さん
10/06/26 16:51:46
>>253
無限大超自然数 ω をひとつ固定する。
P(N) を N<ω と定義すれば、
「N+1以上の自然数nが存在して、P(n)は偽である」
はすべて正しい。

263:132人目の素数さん
10/06/26 17:27:25
wikiのパラドックスの項目。
Wikipedia項目リンク

岩波 数学辞典の内容も引用されているので、ここに書かれていることが
パラドックスの標準的な定義だと思って差し支えないだろう。

そして、>>253は標準的な「パラドックス」とは定義もニュアンスも全然異なる。

264:132人目の素数さん
10/06/27 23:36:37
>>248
メタレベルが無限になって、自然数メタレベルではなくなるからでは?
自己言及も、元はと言えば自分のメタレベルをLとすると自分のメタレベルはL−1となって
自分のメタレベルが一意に定まらないことから来てるはず。
禁則を少し一般化して、「自己言及の文章は命題になるとは限らない」から
「メタレベルが自然数で表せない文章は命題になるとは限らない」に
パラダイムシフトすると解消できるかと。


265:132人目の素数さん
10/06/28 00:09:46
>>264
あーでもそうすると「2は最小の偶数である」とかも命題じゃなくなって
ちょっと困るな。

266:132人目の素数さん
10/06/28 08:26:19
>>261
当然逃げるよ。 
できるかどうかもわからないのに
自分の興味のないものに労力は使いたくない。

267:名無しさん@そうだ選挙に行こう
10/07/10 20:16:26
一辺の長さ1の正方形に
一辺の長さがそれぞれ1/2, 1/3, 1/4,... となるような可算個の正方形たちを
互いに重ならないように敷き詰める方法は?

268:132人目の素数さん
10/07/10 20:27:01
>>267
ない

269:名無しさん@そうだ選挙に行こう
10/07/10 21:23:49
ないな

270:名無しさん@そうだ選挙に行こう
10/07/10 21:46:58
12x12,6,4,3

271:ルンペン猫 ◆ghclfYsc82
10/07/10 21:49:02
>>269
>>270
質問いいかな?




272:名無しさん@そうだ選挙に行こう
10/07/10 21:56:02
12*12*12の立体を1x1x1のブロック組み合わせでフォルトフリーで分割する組み合わせは
いくつ?角だけでつながっていてもいい。

273:ルンペン猫 ◆ghclfYsc82
10/07/10 21:59:38
>>272
質問いいかな?




274:名無しさん@そうだ選挙に行こう
10/07/10 22:05:11
>>267
InkScape(ドローソフト)でお絵かきして1/2から1/13まで詰めてみた感じでは、
かなり適当でも可能とみた。
(1/2)^2 + (1/3)^2 + ・・・ = π^2/6 - 1 = 0.6449...
だから結構スカスカっぽい

275:269 証明はできないけど
10/07/11 02:27:11
>>271
何?

276:名無しさん@そうだ選挙に行こう
10/07/11 07:30:53
>>267
URLリンク(damedao.web.fc2.com)

277:名無しさん@そうだ選挙に行こう
10/07/11 08:01:17
>>276
正解
1*1の正方形には1/2*1/2の正方形を2個、1/4*1/4のを4個、
1/8*1/8のを8個、…と詰め込むことが出来るのを利用する方法ですな

あと
>(1/2)^2 + (1/3)^2 + ・・・ = π^2/6 - 1 = 0.6449...
だから一辺の長さ5/6(面積0.6944....)の正方形にも
267の可算個の正方形達を詰め込める方法があるのかもしれません

278:269
10/07/11 08:45:47
あ、そうか……

一辺が1の正方形も詰め込むこと相当の計算してた
こんなタコミスしてたらそりゃ入るわけねーな…

すまん

279:名無しさん@そうだ選挙に行こう
10/07/11 10:40:06
>>277
過去ログに同じ問題あるはずだよ。
5/6の正方形に詰め込んだ人もいた気がする。

280:名無しさん@そうだ選挙に行こう
10/07/11 13:33:06
>>277
>>276が正解なら
1/2 x 4
1/3 x 9
・・・
でも正解ってことか?
少なくとも問題文に上記の様なのは不可、とは書いてないし

281:名無しさん@そうだ選挙に行こう
10/07/11 16:20:13
>>280
どういう意味?

282:名無しさん@そうだ選挙に行こう
10/07/11 16:24:00
>>281
そのまま。
4分割、9分割、・・・でおk

283:名無しさん@そうだ選挙に行こう
10/07/11 17:29:20
>>282
すまない全く意味が分からない
問題の意味を勘違いしてないか

284:名無しさん@そうだ選挙に行こう
10/07/11 17:46:08
>>283
>それぞれ1/2, 1/3, 1/4,... となるような可算個の正方形
で、同じ大きさの複数の正方形が入れられるなら、
>>276のように1/2 x 2じゃなく、1/2x4でやれるだろう
という事。

1/2、1/3、・・・の正方形のそれぞれの個数の範囲を指定してないから
1/2 x4 でも正解じゃないの?といいたいのだ。

285:名無しさん@そうだ選挙に行こう
10/07/11 18:08:21
パッキング問題だからプログラムだけ証明できればいい。

286:267
10/07/11 18:56:40
>>284
問題は一辺が1/2,1/3,1/4,...の正方形達を一つずつ重ならないように詰め込むこと
>>277は1/2*1/2の正方形を2個、1/4*1/4のを4個、1/8*1/8のを8個、…と
敷き詰める方法を考えたあとに正方形達を小さい正方形に変えれば答えが得られることを
言っているのであって、
1/2*1/2の正方形を2個、1/4*1/4のを4個、1/8*1/8のを8個…と詰め込むこと自体や
ましてや1/2*1/2個の正方形を4個詰め込むことを正解と言っている訳ではありません

287:名無しさん@そうだ選挙に行こう
10/07/11 19:08:04
>>286
最初一つずつかと思ったけど、>>277を見て
複数でも良いのかと思ったのだ。

ちゃんと一つずつって書かないと判らない。

288:名無しさん@そうだ選挙に行こう
10/07/11 23:03:53
>>267は面白い問題おしえて〜な 六問目の912に同じ問題がある
また
URLリンク(web2.incl.ne.jp)
により厳しい設定での解答がある

289:名無しさん@そうだ選挙に行こう
10/07/11 23:19:12
>>288 その愛知の方の解答はすごいですね
あんなのどうやったら思いつくんでしょうか?

290:132人目の素数さん
10/07/11 23:35:19
>>289
リンク先の「31〜∞」みたいな小さい正方形を一斉に詰め込む方法を考えたら
あとは残りの大きい正方形を詰め込む方法を根性で探す

291:132人目の素数さん
10/07/12 08:15:59
>>277
> だから一辺の長さ5/6(面積0.6944....)の正方形にも
> 267の可算個の正方形達を詰め込める方法があるのかもしれません

解答出たけど、昨日から考えてたのが出来たから

URLリンク(damedao.web.fc2.com)

1/24 までの正方形を図のように並べると、
0.23*0.3 の長方形を残せる

1/25 以下の正方形については、
1/n (2^k<n≦2^(k+1)) の正方形を長方形内に図のように
横方向に 2^(k-2) 個並べるようにする

必要なサイズは


= 1/(2^k+1) + 1/(2^k+2) + … + 1/(2^k+2^(k-2))
< ∫[2^k, 2^k+2^(k-2)] dx/x
= ln(5/4) = 0.223

高さ
< 1/25 + 1/29 + (1/8)*(1/4+1/5+1/6+1/7) + (1/16)*(1/4+1/5+1/6+1/7) + …
= 1/25 + 1/29 + (1/4)*(1/4+1/5+1/6+1/7)
= 0.264

だから長方形に納まる

292:132人目の素数さん
10/07/12 09:07:03
どれも敷き詰めになってない

293:132人目の素数さん
10/07/12 14:37:29
ことばの使い方に終始したいなら別の板行った方がいいんじゃないかね

294:132人目の素数さん
10/07/12 16:16:37
GREG MARTIN て方の論文
「COMPACTNESS THEOREMS FOR GEOMETRIC PACKINGS」
URLリンク(citeseerx.ist.psu.edu)
では、

任意の正数εに対して
(π^2/6 - 1) + ε の面積をもつ、ある長方形に

>一辺の長さがそれぞれ1/2, 1/3, 1/4,... となるような可算個の正方形たちを
>互いに重ならないように敷き詰める方法

が存在する事を証明しています。
私には英語以前に数式が理解できそうもありませんでした。
図も一切ありません。

295:132人目の素数さん
10/07/12 22:13:09
>>292
つまり
スカスカになる
が正解、ってこと

296:132人目の素数さん
10/07/15 23:51:59
I=[0,1]としf:I→Iとg:I→Iが連続関数でx∈I→f(g(x))=g(f(x))のとき
f(x)=g(x)となるx∈Iがあることを証明せよ

297:132人目の素数さん
10/07/16 00:28:47
f (x)=g(x)なるx∈Iが存在しないとして矛盾を導く。

任意のxに対してf (x)≠g(x)ならば、中間値の定理から、
常にf (x)<g(x)であるか、あるいは常にf (x)>g(x)である。
常にf (x)<g(x)としてよい。

f :I → Iだから、f は不動点を必ず持つ。不動点の1つをx0と置く。
f (x0)=x0 だから、f (g(x0))=g(f (x0))=g(x0)となる。よって、
g(x0)もまたf の不動点である。そこで、x1=g(x0)と置く。
以下、同様の作業を繰り返して不動点xnを作ると、
(1) f (xn)=xn
(2) x_{n+1}=g(xn)
が成り立つ。f (x)<g(x)よりxn=f (xn)<g(xn)=x_{n+1}となるので、
xnは単調増加である。また、xn∈Iだから、xnは上に有界である。
よって、xnはあるx∈Iに収束する。(1),(2)でn→∞として、
f (x)=x及びx=g(x)を得るので、g(x)=f (x)となり、矛盾する。

298:132人目の素数さん
10/07/16 05:17:22
正解です


最新レス表示
スレッドの検索
類似スレ一覧
話題のニュース
おまかせリスト
▼オプションを表示
暇つぶし2ch

4765日前に更新/96 KB
担当:undef